LSAT and Law School Admissions Forum

Get expert LSAT preparation and law school admissions advice from PowerScore Test Preparation.

 Administrator
PowerScore Staff
  • PowerScore Staff
  • Posts: 8917
  • Joined: Feb 02, 2011
|
#35163
Complete Question Explanation

Strengthen—CE. The correct answer choice is (A)

Your task in this Strengthen question is to select the answer choice that most supports the scientist’s
conclusion. The scientist’s argument proceeds:

..... Premise: ..... physicists claim their system of careful scientific peer review prevents
..... ..... ..... ..... scientific fraud in physics effectively

..... Premise: ..... but, biologists claimed the same thing 20 years ago, and the turned out to be
..... ..... ..... ..... wrong

..... Premise: ..... biologists have since greatly enhanced their discipline’s safeguards against
..... ..... ..... ..... scientific fraud, preventing further major incidents

..... Conclusion: ..... it would be conducive to progress in physics if physicists were to do the same
..... ..... ..... ..... thing

Your prephrase is that the correct answer will support the scientist’s conclusion that enhancing
safeguards against scientific fraud would be conducive to progress in physics. Specifically, a gap in
this causal argument is that prior to the conclusion, the argument mentions nothing about progress,
either in reference to biology or physics. So, the correct answer will likely connect enhancing the
safeguards against scientific fraud in physics to progress in physics.

The incorrect choices will not support the conclusion, either because they have no effect on the
conclusion or weaken it.

Answer choice (A): This is the correct answer choice. The premise regarding biology established
that enhancing the safeguards prevented further major incidents. Assuming that physics and
biology are sufficiently similar as academic disciplines, it is reasonable to conclude that enhancing
safeguards in physics will also lead to the prevention of further major incidents in physics. This
choice provides an additional causal relationship that connects major incidents to the idea of
progress. If it is the case that major incidents of scientific fraud in a scientific discipline are
deleterious to progress in that discipline, then preventing such major incidents would remove the
deleterious effect, thus being conducive to progress. Do not mistake this to mean removing major
incidents will cause progress. Rather, it will create an environment in which progress may occur.

Answer choice (B): This information supports the premise that increasing safeguards in chemistry
prevented further major incidents, but does nothing to support the conclusion drawn from that
premise.

Answer choice (C): This choice does not affect the argument, which discussed preventing further
major incidents of fraud, and did not deal with completely preventing scientific fraud.

Answer choice (D): This choice has no effect on the conclusion, because the comparison it evokes is
not material to the conclusion.

Answer choice (E): This answer choice would tend to undermine the conclusion that increased
safeguards in physics would produce any results.
 lilyzhang
  • Posts: 2
  • Joined: Oct 17, 2016
|
#29808
Hello,

This question really got me think when I was doing the PT but I still got it wrong... :(

I know that this is strengthen question and I chose C instead of A. I chose C thinking that if no system of careful peer review is completely effective, enhancing it will definitely help make progress in Physics. But when I was reviewing it, I looked at A and thought it would be also be a good answer. I don't see what makes A better than C. It is because it explicitly mentions progress? Is this questions more of a Justify question?

Thanks in advance!

Lily
 David Boyle
PowerScore Staff
  • PowerScore Staff
  • Posts: 836
  • Joined: Jun 07, 2013
|
#29821
lilyzhang wrote:Hello,

This question really got me think when I was doing the PT but I still got it wrong... :(

I know that this is strengthen question and I chose C instead of A. I chose C thinking that if no system of careful peer review is completely effective, enhancing it will definitely help make progress in Physics. But when I was reviewing it, I looked at A and thought it would be also be a good answer. I don't see what makes A better than C. It is because it explicitly mentions progress? Is this questions more of a Justify question?

Thanks in advance!

Lily


Hello Lily,

It is a Strengthen question, as they ask for what "is most strongly supported".
Answer C would not be helpful, as we aren't really thinking that systems of peer review are perfect anyway. Answer A is much better, since it links fraud to progress.

Hope this helps,
David
 cardinal2017
  • Posts: 19
  • Joined: Oct 23, 2016
|
#33937
I have a different explanation on the correct answer.

I actually chose (C) as well initially, but then (C) doesn't make the strongest case out for the conclusion since it links the gap between biology and physics while it doesn't address the new information in the conclusion: the progress thing there.

((A) is the answer bx it is what "most strongly supports" the conclusion.

It addresses the gap as in mentioning and connecting "major incidents of sci fraud"(in the Premise part) with "progress in the discipline" of physics (in the Conclusion part).

Put another way, the (A) links the premise and the conclusion, and thereby strengthening the argument as kind of a sufficient assumption.

I hope this works for you Lily and any others who might struggle with the question.
 Robert Carroll
PowerScore Staff
  • PowerScore Staff
  • Posts: 1787
  • Joined: Dec 06, 2013
|
#33961
cardinal,

You're correct that the problem with this conclusion is that we have no link between the premises and progress. The premises make a good, if not perfect, case that doing what biology did would reduce fraud. What this has to do with progress is not explained. Thus, the correct answer will show that reducing fraud is good for progress, as answer choice (A) does, by saying that fraud hinders progress (so reducing fraud would enhance progress).

Robert Carroll
 mN2mmvf
  • Posts: 113
  • Joined: Jul 06, 2017
|
#39108
Can you help me understand why (A) supports the argument? I took the stimulus to say that physicists already acknowledge that fraud is a problem, which is why they have a "system of careful peer review" that they claim is working effectively. There's no evidence presented that, in physics, it isn't working effectively. (Just that, in biology, there's a system that is claimed should be used in physics too.) So what's the point in again assuming that "major incidents of fraud...are deleterious to progress"? Physicists already know that.

Instead, I was trying to strengthen the argument not by proving that fraud is a problem, but by strengthening the argument that the biology method should be adopted even though physics already has one of its own. That, to me, is the heart of the argument, with very little support provided. Thus, I chose (B), thinking that, even if the current system for physics was working well, it may not be working as well as the one biology has, if the system biology has is just about perfect. That's what (B) says.
 nicholaspavic
PowerScore Staff
  • PowerScore Staff
  • Posts: 271
  • Joined: Jun 12, 2017
|
#39842
Hi mN2,

We are going to have to identify the conclusion in order to support it as the question stem calls upon us to do. Here, as we note above, the conclusion is that it would be conducive to progress in physics if physicists were to enhance safeguards. It appears as though you may have struggled with effectively identifying that conclusion. In most Strengthening questions, we are being asked to identify, isolate and then strengthen argument's conclusions. It's also important to note that "progress" is only mentioned in the conclusion which is why Answer (A) is correct. A classic wrong answer such as Answer Option (B) may strengthen a premise, but ultimately that will not strengthen the conclusion. Accordingly, try to always identify the conclusion to better understand the argument that you are trying to support.

Thanks for the great question.
 Sophia123
  • Posts: 43
  • Joined: Mar 20, 2017
|
#49282
Hi,

For this question, I am having a hard time seeing why this would be classified as a strengthen question as opposed to a justify the conclusion question? It appears to me with the classic justify approach, it is easier to identify the rogue element in the conclusion "conducive to progress in physics" in order to get to the correct answer. Can someone help me understand why this would be categorized as a strengthen as opposed to a justify?

Thanks in advance!

Best,
Sophia
 Adam Tyson
PowerScore Staff
  • PowerScore Staff
  • Posts: 5153
  • Joined: Apr 14, 2011
|
#49335
It all comes down to the language in the stimulus, Sophia, and the crucial term here is "most." A true Justify question doesn't do anything to lessen the amount of justification below 100%! When you are trying to justify (prove) a conclusion, there is no "most" or "helps" about it - it's either all the way or it isn't.

Here, we aren't asked to prove that the conclusion is true, but only to identify which answer does the most to strengthen it. Typically that would mean that the four wrong answers will not strengthen it at all, while the correct one will at least strengthen it a little, perhaps a lot. The correct answer might justify the conclusion, but it doesn't have to. That's why this one is only a Strengthen question - we don't need to prove anything, just help.

The approach doesn't dictate the question type, but rather the question type dictates the approach.

Get the most out of your LSAT Prep Plus subscription.

Analyze and track your performance with our Testing and Analytics Package.